LSAT and Law School Admissions Forum

Get expert LSAT preparation and law school admissions advice from PowerScore Test Preparation.

 sarae
  • Posts: 80
  • Joined: Aug 10, 2013
|
#11532
I was a little confused on this stimulus/answer choices. please explain why E is correct!

thanks!
 Ron Gore
PowerScore Staff
  • PowerScore Staff
  • Posts: 220
  • Joined: May 15, 2013
|
#11565
Hi, Sara, thanks for your question.

This is a Method of Reasoning, Flaw in the Reasoning question. Your task is to select the answer choice that best describes the flaw in the argumentation in the stimulus.

To identify the flawed logic, you first have to focus in on the conclusion, which is the first sentence of the stimulus. You may have been confused by the stimulus because it provides a conditional relationship in the second sentence, but then the conclusion does not result from some application of that relationship, for example a Mistaken Reversal or a Mistaken Negation.

Instead, the conclusion results from the premise that apparently altruistic behavior can be understood as merely self-interested. From the idea that behavior can be understood as self-interested rather than altruistic, the argument infers that there is no behavior that is actually altruistic. This shift from the possibility that the behavior can be understood in a certain way to the conclusion that the behavior must then, in fact, be that way is not supported by any premise.

Answer choice (E) describes this flaw, assuming that because behavior can be interpreted as self-interested, then it must actually be self-interested, i.e., not altruistic.

Please let me know if I can help you further.

Thanks!

Ron
 persde
  • Posts: 8
  • Joined: Oct 10, 2017
|
#41997
I chose answer choice (B) on this one. Is this a Shell answer? I can see why answer choice (E) is correct but I don't fully understand why answer (B) is incorrect. Can you please explain?

Thank you!
 Claire Horan
PowerScore Staff
  • PowerScore Staff
  • Posts: 408
  • Joined: Apr 18, 2016
|
#42062
Hi Persde,

The conclusion of this stimulus is "There is no genuinely altruistic behavior." Answer (B) says that the argument "infers that behavior is altruistic...", which is the opposite of the conclusion.

For a flaw in the reasoning answer choice to be correct, it must 1) accurately describe what is happening in the stimulus and 2) stimulus must contain flawed reasoning. So remember always to check that the answer choice actually describes the argument correctly.

And since you mentioned shell game, that is more like, for example, in parallel reasoning questions when the stimulus and an answer choice are both about cats, but the logic used is nothing alike. They look similar only superficially because they involve the same topic.

Good luck with your studying!
 lbevins
  • Posts: 3
  • Joined: Jul 18, 2018
|
#48792
During the practice test I chose E but after blind review I had selected B. My reasoning was based on the wording "Behavior that appears to be altruistic..." - just because behavior appears to be altruistic doesn't mean that is is altruistic. Can someone explain why B is wrong more thoroughly so I can better understand the reasoning? Thank you
 Malila Robinson
PowerScore Staff
  • PowerScore Staff
  • Posts: 296
  • Joined: Feb 01, 2018
|
#48856
Hi lbevins,
Since this is a Flaw Question you need to be able to point to a place in the stimulus that the answer choice refers to. B states: "illicitly infers that behavior is altruistic merely because it seems altruistic" But the argument is saying that just because something seems altruistic it doesn't mean that it is (which goes against Answer B), and then it gives the reasons why something may seem altruistic but not really be altruistic.
Hope that helps!
-Malila
 SammyWu11201
  • Posts: 29
  • Joined: Jun 29, 2020
|
#81422
I don’t understand why A is wrong and after watching the video, I don’t get why JY is saying that it is wrong because it is backwards. The stimulus is saying that everyone needs a sufficient amount of self-esteem and it is out of that need for self-esteem that they are being altruistic. In my mind, A is the correct answer because it is flawed that the stimulus presupposes that because what if the person already has an adequate amount of self-esteem and therefore doesn’t need to bolster it with altruistic behaviors but is still doing those altruistic actions solely to be altruistic?
User avatar
 KelseyWoods
PowerScore Staff
  • PowerScore Staff
  • Posts: 1079
  • Joined: Jun 26, 2013
|
#81502
Hi Sammy!

Careful with how you interpret the argument here. Let's break it down:

Conclusion: There is no genuinely altruistic behavior.
Premise: Behavior that appears to be altruistic can be understood as being motivated by a clearly self-interested motivation (believing oneself to be useful and needed).

All logical flaws boil down to this: the premises given do not fully prove the conclusion as stated. To figure out the specific flaw, always specifically identify the conclusion and the premises and ask yourself why the premises given do not fully prove the conclusion as stated.

Answer choice (A) states that the argument "presupposes that anyone who is acting out of self-interest is being altruistic." But the argument does not do this. Answer choice (A) describes an argument that goes something like this: "Bob is acting out of self-interest. Therefore, Bob is being altruistic." But the conclusion in our argument is that genuine altruistic behavior does not exist. So the argument in our stimulus is not presupposing that someone acting out of self-interest is being altruistic. Rather, the argument is saying that seemingly altruistic behavior could be motivated by self-interest, and so truly altruistic behavior does not exist.

The correct answer in a Flaw question has to describe what actually occurs in the argument while also describing what specifically explains why the premises do not prove the conclusion. So always make sure that you have clearly analyzed the premises and the conclusion to make sure the answer choice is an adequate description of the argument and its flaw.

Hope this helps!

Best,
Kelsey
User avatar
 CJ12345:
  • Posts: 56
  • Joined: May 25, 2023
|
#103749
Hi, Powerscore,
I had a hard time with this Q because the stimulus present a conditional reasoning. Thus, I am trying to see how it reaches the conclusion based on the conditional reasoning (everyone needs sufficient self-esteem --> believe oneself to be useful and needed; behavior that appears to be altruistic can be understand as = believe oneself to be useful and need = self-interested motivation). I was stuck into this FL and saw that the author might make a mistake that believe oneself to be useful and needed does not lead to sufficient self-esteem, and I was trying to find an AC that talks about this flaw (sort of like sufficient necessary flaw). However, ACs have nothing to do with the FL. How can I quickly realize that I should not rely on the FL when doing questions next time? Why the conditional reasoning in this case is not useful at all when doing Q?
 Rachael Wilkenfeld
PowerScore Staff
  • PowerScore Staff
  • Posts: 1419
  • Joined: Dec 15, 2011
|
#103801
Hi CJ

A few things here.

First, I'd note that the conditionality isn't something unhelpful here. It's very helpful in thinking through the flaw. If we look at the conditionality, it states that self-esteem REQUIRES believing oneself useful and needed. The flaw comes in building from that conditional. Just because self-esteem requires that you believe yourself useful and needed doesn't mean that anything that COULD make you feel useful/needed is done to build self-esteem. That's where the self-interested part comes in. The author mistakenly assumes because self-esteem requires something, people will only use that thing to support their self-esteem. It's a sort of mistaken reversal in concept if not in the typical language.

Second, I'd say no matter what the type of reasoning in the stimulus, when there is an argument, you need to focus on the space between the premises that you can believe, and the conclusion that you don't believe. Where does the author go wrong? What does that space between the premises and conclusion look like? That's where the flaw will be located. In this case, you see the conditionality in the premises, but the conclusion draws the wrong inference from that conditionality.

Hope that helps!

Get the most out of your LSAT Prep Plus subscription.

Analyze and track your performance with our Testing and Analytics Package.